Acing the SQE/SQE1 Sample

From Wikibooks, open books for an open world
Jump to navigation Jump to search

FLK1 Answers and Explanations[edit | edit source]

Question 1[edit | edit source]

Contract Law: Formation: offer and acceptance

https://sqe.sra.org.uk/exam-arrangements/assessment-information/sqe1-sample-questions/flk1/question1

The best answer is B.

The website designers made an offer through their leaflet by advertising their website design packages and including their standard terms and conditions, which contained a limitation clause. However, this offer was not accepted by the architect merely by asking for a quotation. This action only amounted to an invitation to treat, which is an invitation to negotiate or start a contract.

The architect made a counter offer by sending a letter with his own standard terms and conditions, which did not contain a limitation clause. However, by signing and returning the tear-off slip stating that he accepted the quotation on the website designers’ standard terms and conditions, the architect accepted the website designers' offer, and the contract was formed on their terms.

Therefore, option B is the most accurate statement because the quotation constituted an offer which the architect accepted on the website designers’ standard terms and conditions.

In English contract law, a limitation clause is a provision in a contract that seeks to limit the liability of one or both parties in the event of a breach of contract or other loss or damage.

For example, a limitation clause may limit the amount of damages that a party can claim, or exclude certain types of losses or damages altogether. It may also limit the time within which a party can bring a claim, or require that certain conditions be met before a claim can be made.

Limitation clauses are often used in commercial contracts, where the parties may wish to limit their exposure to potential losses or liabilities. They are also commonly found in consumer contracts, such as insurance policies or standard terms and conditions, where the supplier may seek to limit its liability to the consumer in the event of a breach of contract.

However, limitation clauses are subject to certain legal requirements and limitations. For example, under the Unfair Contract Terms Act 1977, limitation clauses that seek to exclude or limit liability for death or personal injury resulting from negligence are generally unenforceable.

Similarly, a limitation clause may be unenforceable if it is found to be unfair or unreasonable, or if it is not sufficiently clear and specific to be enforceable. Therefore, it is important for parties to carefully draft limitation clauses to ensure that they are legally enforceable and do not run afoul of these limitations and requirements.

Question 2[edit | edit source]

Business Law and Practice: Insolvency (corporate and personal): order of priority for distribution to creditors.

https://sqe.sra.org.uk/exam-arrangements/assessment-information/sqe1-sample-questions/flk1/question2

The correct answer is C.

The priority of competing security interests in a company's assets is determined by the rules of priority established under the law of secured transactions. In this case, there are two competing security interests over the company's assets: the trade supplier's floating charge and the bank's floating charge.

Under the general rule of priority, a later security interest will have priority over an earlier one if it is created in good faith, for valuable consideration, and without notice of the earlier interest. However, if the earlier interest is properly registered, it will generally take priority over any later interest, regardless of when it was created or whether the later holder had notice of it.

In this case, the trade supplier's floating charge was created earlier than the bank's floating charge, but it was not registered at Companies House. The bank was aware of the existence of the trade supplier's charge, but it was not registered at the time the bank created its own charge.

The failure to register the trade supplier's charge means that it is void against the administrator and the bank. This means that the administrator and the bank are not bound by the trade supplier's charge and can enforce their own security interests over the company's assets. As a result, the bank's floating charge takes priority over the trade supplier's charge, even though it was created later in time.

In summary, the failure to register the trade supplier's floating charge means that it is void against the administrator and the bank. This allows the bank's floating charge to take priority over the trade supplier's charge, even though it was created later in time.

In English company law, a floating charge is a type of security interest that can be taken out over a company's assets to secure a loan or other form of credit. Unlike a fixed charge, which attaches to specific assets of the company (such as property or machinery), a floating charge is a charge over a group of assets that are subject to change over time.

The assets covered by a floating charge may include inventory, accounts receivable, and other assets that are subject to fluctuation in value and turnover. The charge "floats" over these assets, meaning that the company can continue to use and dispose of them in the ordinary course of business, subject to certain restrictions set out in the charge agreement.

A floating charge allows a lender to secure a loan without taking possession of specific assets, which can be beneficial for both the lender and the company. For the lender, a floating charge provides security over the company's assets while allowing the company to continue to operate and generate income. For the company, a floating charge can provide more flexibility in managing its assets, as it can continue to use and dispose of the assets covered by the charge without seeking permission from the lender.

However, there are limitations on the use of floating charges in company law. For example, a floating charge may be invalid if it is created to secure past debts, rather than future debts. Additionally, if a company enters into administration or liquidation, a floating charge will "crystallize" into a fixed charge, which means that the lender will no longer be able to use the assets covered by the charge.

Overall, floating charges are an important tool in the financing of businesses in the UK, providing a flexible means of securing credit while allowing companies to continue to operate and manage their assets.

In English company law, a debenture is a type of document that creates a fixed or floating charge over a company's assets as security for a loan or other form of credit. When a company enters into a debenture with a bank, it is essentially agreeing to use its assets as collateral for the loan.

A debenture can take many forms, but it typically sets out the terms of the loan, the amount of the loan, the interest rate, and the repayment terms. The debenture will also specify the assets that are being used as collateral for the loan, whether they are fixed or floating assets.

If the debenture creates a fixed charge, it means that the assets being used as collateral are specific and clearly defined, and cannot be sold or otherwise disposed of without the lender's consent. Examples of fixed assets might include property or machinery. If the debenture creates a floating charge, it means that the assets being used as collateral are not specific and may change over time. Examples of floating assets might include inventory, accounts receivable, or other assets that are subject to fluctuation.

Entering into a debenture with a bank can be an important means for a company to secure financing, but it also comes with certain risks. If the company defaults on the loan, the bank may take possession of the assets used as collateral to recover the debt. Therefore, it is important for companies to carefully consider the terms of the debenture and ensure that they are able to meet their obligations under the loan agreement.

Question 3[edit | edit source]

Dispute Resolution: Disclosure and inspection: privilege and without prejudice communications

https://sqe.sra.org.uk/exam-arrangements/assessment-information/sqe1-sample-questions/flk1/question3

The correct answer is D.

Litigation privilege protects confidential communications between a client and their lawyer or third parties for the dominant purpose of obtaining or giving legal advice in connection with actual or anticipated litigation.

In this case, the two-page letter was written by an independent financial adviser with the sole purpose of giving advice about drafting the letter before claim in the present litigation. Therefore, it was created for the dominant purpose of obtaining legal advice in connection with the present litigation, and it is likely to be protected by litigation privilege.

The fact that the letter was found in the office of the claimant’s managing director and is in the possession of the claimant’s solicitor does not affect the claimant's right to withhold inspection of the letter under litigation privilege.

Option A is incorrect because control is not a relevant factor for determining whether litigation privilege applies.

Option B is incorrect because legal advice privilege only applies to confidential communications between a client and their lawyer for the purpose of giving or receiving legal advice.

Option C is incorrect because proportionality is a factor to be considered in granting or denying inspection of a document, but it is not a basis for claiming a right to withhold inspection.

Option E is incorrect because the timing of the letter does not affect the applicability of litigation privilege.

Question 4[edit | edit source]

Business Law and Practice: Legal personality and limited liability.

https://sqe.sra.org.uk/exam-arrangements/assessment-information/sqe1-sample-questions/flk1/question4

Option C is correct. In this situation, the contract for the supply of cycling clothing was signed by the cycling enthusiast in his own name, but on behalf of the company before it was formally incorporated. Therefore, the contract is not legally binding on the company because the company did not exist at the time the contract was signed.

Until a company is incorporated, it has no legal personality, rights or obligations. Therefore, any contracts entered into before incorporation are not binding on the company, and the promoter who signed the contract will be personally liable for any obligations arising from the contract.

In this case, because the contract was signed by the cycling enthusiast in his own name, on behalf of the company, the obligation to perform the contract rests with the enthusiast personally rather than the company. The benefit of the contract will therefore also reside with the enthusiast alone, and not with the company or its shareholders.

In order to make the contract binding on the company, the enthusiast would need to sign a new contract in the name of the company after it has been formally incorporated. This would ensure that the company, as a separate legal entity, is bound by the terms of the contract and can enjoy its benefits.

Question 5[edit | edit source]

Public Law: judicial review: standing

https://sqe.sra.org.uk/exam-arrangements/assessment-information/sqe1-sample-questions/flk1/question5

Option C is correct.

The cousin cannot bring legal proceedings as the woman’s representative under the HRA.

Under the HRA, a victim of an infringement of their Convention rights may bring proceedings in court against a public authority. However, the HRA does not provide for representative actions, except in specific cases where the victim is dead or lacks capacity. Option A is incorrect because family representatives are not specifically granted standing under the HRA. Option B is incorrect because although anonymity is a serious concern for the victim, this alone is not sufficient to allow someone else to bring proceedings on her behalf. Option C is the correct answer because only a victim of an infringement can bring an action under the HRA. The cousin is not a victim of the infringement, and therefore cannot bring proceedings on behalf of the woman. Option D is incorrect because the court's satisfaction that the representative is able to meet the expenses of the proceedings is not sufficient to allow someone else to bring proceedings on behalf of the victim. Option E is incorrect because the provision for representative actions under the HRA applies only in limited circumstances where it is not reasonably practicable for the victim to bring separate actions, such as where there are numerous individuals making identical claims.

Question 6[edit | edit source]

Business Law and Practice: Corporate governance and compliance:rights, duties and powers of directors and shareholders of companies

https://sqe.sra.org.uk/exam-arrangements/assessment-information/sqe1-sample-questions/flk1/question6

Option D is correct.

Yes, because the transaction involves the sale and purchase of land whose value exceeds 10% of the company’s asset value, which triggers the requirement for shareholder approval under the Companies (Model Articles) Regulations 2008 (unamended). This is because the sale of a substantial non-cash asset such as land requires the approval of the company's shareholders under the regulations, and the value of the land being sold exceeds 10% of the company's net assets. The fact that the proposed purchaser is known to the directors, and that they are also shareholders, is not relevant to the requirement for shareholder approval.

Question 7[edit | edit source]

Contract Law: Remedies:damages

https://sqe.sra.org.uk/exam-arrangements/assessment-information/sqe1-sample-questions/flk1/question7

C is correct.

The most likely remedy that the court will award in this situation is the cost of cure. This is a form of damages that requires the supplier to pay the reasonable cost of replacing the tiles with tiles that include the correct border design showing the Greek flag. The aim of this remedy is to put the owner of the restaurant in the position they would have been in if the contract had been performed correctly, by giving them the tiles that they originally ordered and paid for.

Question 8[edit | edit source]

Dispute Resolution:

https://sqe.sra.org.uk/exam-arrangements/assessment-information/sqe1-sample-questions/flk1/question8

The correct answer is C. The Limitation Act 1980 sets out time limits within which legal claims must be brought. The time limit for breach of contract claims is generally six years from the date of the breach. In this case, the alleged breach of contract by the plumber took place in June 2015, and the claim was brought in March 2021, within the six-year time limit. When a party to a legal claim dies, the claim does not automatically come to an end. The court has the power to allow the claim to continue against the deceased party's personal representatives, who are responsible for dealing with the deceased person's assets and liabilities. This is done by substituting the personal representatives for the deceased party in the claim. In this case, the plumber died after the claim was brought but before it was resolved. As a result, the claimant applied to substitute the plumber's personal representatives for the deceased plumber in the claim, so that the claim could continue. The court has the power to order a new party to be substituted in a legal claim if the relevant limitation period was current when the proceedings were started. In this case, the claim was brought within the six-year time limit for breach of contract claims, and therefore the court has the power to allow the claimant to substitute the plumber's personal representatives as the defendant in the claim.

Question 9[edit | edit source]

Contract Law:

https://sqe.sra.org.uk/exam-arrangements/assessment-information/sqe1-sample-questions/flk1/question9

The correct answer is B.

B is correct. There is a contract between the man and the woman because sufficient consideration has been promised. Consideration is a key element in forming a contract, and it refers to something of value that is given in exchange for something else. In this case, the man promised to sell the bicycle to the woman for £25, and the woman promised to pay that amount. This exchange of promises is sufficient consideration to form a contract. Although the man later discovered that the bicycle was worth much more than he initially thought, that does not change the fact that a contract was already formed between the two parties. The man cannot unilaterally cancel the contract simply because he made a mistake about the value of the bicycle. Therefore, if the woman still wants to buy the bicycle for £25, she can hold the man to his promise and force him to sell it to her. However, if the woman agrees to release the man from the contract, then it would be terminated.

Question 10[edit | edit source]

The Legal System: The courts:court hierarchy, the appeal system and jurisdiction

https://sqe.sra.org.uk/exam-arrangements/assessment-information/sqe1-sample-questions/flk1/question10

The correct answer is D.

The House of Lords has been replaced by the Supreme Court as the highest court in the UK. Therefore, option C is no longer correct. Under the current legal system, the Court of Appeal and the Supreme Court have the power to grant permission to appeal. The Court of Appeal may grant permission to appeal against its own decision, either at the hearing or later, if an application is made. The Supreme Court may grant permission to appeal against a decision of the Court of Appeal or, in exceptional cases, against a decision of a lower court, if it considers that the case raises an important point of principle or practice, or that there is some other compelling reason for the appeal to be heard. Therefore, the correct answer is D, the Court of Appeal and the Supreme Court have the power to grant permission to appeal.

Question 11[edit | edit source]

Tort Law: Nuisance:public and private nuisance

https://sqe.sra.org.uk/exam-arrangements/assessment-information/sqe1-sample-questions/flk1/question11

The correct answer is B.

The cause of action the woman should pursue in tort is B. Public nuisance. Public nuisance is an unreasonable interference with a right that is common to the public. In this case, the man's farmers market is causing the village to become very busy on Thursdays, which is affecting the woman's ability to deliver her goods and resulting in a loss of trade. This interference with the woman's right to carry out her business activities is a common issue affecting the public. Private nuisance, on the other hand, refers to an unreasonable interference with an individual's use or enjoyment of their property. In this case, the woman is not complaining about any interference with her property, but rather the impact on her business activities.

Rylands v Fletcher is a strict liability tort that arises when there is an escape of a dangerous substance from a defendant's land, causing damage to a claimant's property. Negligence and trespass to land are also not applicable to this scenario, as they do not adequately address the issue of public interference that is affecting the woman's business.

Therefore, the most appropriate cause of action for the woman in this case is public nuisance.

Question 12[edit | edit source]

Business Law and Practice:

https://sqe.sra.org.uk/exam-arrangements/assessment-information/sqe1-sample-questions/flk1/question12

The correct answer is D.

The liability to pay Capital Gains Tax (CGT) on the gain realised on the sale of the office premises will be on the individual partners and not on the firm as a separate legal entity. The liability to pay CGT will be in accordance with their profit sharing ratios for capital gains, which are based on their capital contributions. Therefore, the senior partner will be liable to pay CGT on 50% of the gain, the managing partner on 30% of the gain and the junior partner on 20% of the gain. This is because they share capital profits in accordance with their capital contributions. The fact that they share income profits equally does not affect their liability for CGT.

Question 13[edit | edit source]

Taxation: https://sqe.sra.org.uk/exam-arrangements/assessment-information/sqe1-sample-questions/flk1/question13

The correct answer is C.

The trading loss of £45,000 made in the second accounting period can be set off against the chargeable gain of £75,000 from the first accounting period because the chargeable gain occurred within the 12 month period ending immediately before the accounting period in which the trading loss was incurred. This is known as the "same accounting period" rule. Option A is incorrect because the fact that no trading or capital losses were incurred in the first accounting period is not relevant to the application of the same accounting period rule. Option D is also incorrect because the company does not need to have been carrying on business for a full 12 month period before the accounting period in which the trading loss was incurred for the same accounting period rule to apply.

Question 14[edit | edit source]

Business Law and Practice:

https://sqe.sra.org.uk/exam-arrangements/assessment-information/sqe1-sample-questions/flk1/question14

The correct answer is D.

The shareholders’ resolution to adopt the New Articles and the New Articles must be filed with the Registrar of Companies. This is because, under section 22 of the Companies Act 2006, any amendment to a company’s articles of association must be evidenced by a copy of the resolution adopting the amendment, and a copy of the articles as amended. The prescribed fee must also be paid. Board minutes proposing the changes to the Model Articles and the New Articles may be kept as internal records but do not need to be filed with the Registrar.

Question 15[edit | edit source]

Legal System: https://sqe.sra.org.uk/exam-arrangements/assessment-information/sqe1-sample-questions/flk1/question15

The correct answer is D.

The golden rule is a method of statutory interpretation where the judge departs from the literal meaning of the words used in a statute to avoid an absurd or unjust result. In this scenario, the judge looked at the natural and ordinary meaning of the words used in the relevant statute, but found that this interpretation would lead to an absurd meaning. Therefore, the judge interpreted the words in a different way to avoid this absurdity, which is consistent with the application of the golden rule. Option A, the literal rule, would require the judge to give effect to the natural and ordinary meaning of the words used in the statute, even if this leads to an absurd result. Option B, the mischief rule, involves interpreting a statute to give effect to the intention of Parliament by considering the problem the statute was intended to remedy, but this does not appear to be relevant to the scenario described. Option C, the extrinsic evidence rule, involves using external sources such as parliamentary debates or historical context to interpret a statute, but there is no mention of such sources being used in this scenario. Option E, the intrinsic evidence rule, involves looking at the language and structure of the statute itself to interpret it, but this does not seem to be the method used by the judge in this scenario.

Question 16[edit | edit source]

Public Law:

https://sqe.sra.org.uk/exam-arrangements/assessment-information/sqe1-sample-questions/flk1/question16

The correct answer is B.

This is the general principle of the British constitutional law, that when there is a conflict between a statute and a prerogative power, the statute prevails. In this case, although the government is exercising its prerogative powers, the compensation scheme created by Parliament in the legislation has the force of law and the government cannot avoid its obligation to pay compensation by relying on its prerogative powers. The affected occupiers of the buildings can rely on this legislation to claim compensation for any costs and losses incurred while the buildings are under the government's control.

Question 17[edit | edit source]

Ethics

https://sqe.sra.org.uk/exam-arrangements/assessment-information/sqe1-sample-questions/flk1/question17

The best answer is E.

The solicitor should write to the client to inform him about the cost of the expert and ask for his instructions because the original estimate is no longer accurate. The solicitor has a duty to keep the client informed of the likely costs of the case and any material changes to that estimate. The addition of the expert's fees is a material change, which means that the original estimate is no longer accurate. The solicitor must therefore inform the client of the new estimate and seek instructions before incurring any additional costs. Therefore, option E is the correct answer.

Question 18[edit | edit source]

Public Law:

https://sqe.sra.org.uk/exam-arrangements/assessment-information/sqe1-sample-questions/flk1/question18

The correct answer is A.

Under Article 3 of the Convention, no one shall be subjected to torture or to inhuman or degrading treatment or punishment. The High Court has accepted the man's assertion that evidence will be used against him which has been obtained by the use of torture. If the man were deported to his country of origin, he would be at risk of being subjected to torture or inhuman or degrading treatment or punishment. As a signatory to the Convention, the UK is obliged to secure the Convention rights for everyone within its jurisdiction, including non-UK nationals. Therefore, the High Court would not uphold the deportation order.

Question 19[edit | edit source]

Legal Services:

https://sqe.sra.org.uk/exam-arrangements/assessment-information/sqe1-sample-questions/flk1/question19

E is the best answer.

Under section 19 of the Financial Services and Markets Act 2000 (FSMA 2000), a person cannot carry out regulated activities in the UK unless that person is authorised or exempt. This is known as the general prohibition. There are exclusions from the general prohibition which apply to several specific kinds of activity and which are additional to those for regulated activities generally. Article 70 exclusion which applies to the regulated activities of dealing as principal or agent, arranging deals and giving advice in connection with the purchase or sale of shares in a company. There are conditions attached to the use of the exclusion: either the shares must consist of 50% or more of the voting shares in the company or the shares -together with any shares already held by the purchaser - consist of 50% of the voting shares, and each of the parties is a company, partnership, single individual or a group of connected individuals; or the object of the transaction can reasonably be regarded as the acquisition of day to day control of the company's affairs.

Question 20[edit | edit source]

Dispute Resolution:

https://sqe.sra.org.uk/exam-arrangements/assessment-information/sqe1-sample-questions/flk1/question20

The correct answer is D.

The burden lies with the claimant to prove the allegation on the balance of probabilities. In civil cases, the standard of proof is on the balance of probabilities.

Question 21[edit | edit source]

Tort Law:

https://sqe.sra.org.uk/exam-arrangements/assessment-information/sqe1-sample-questions/flk1/question21

The correct answer is A.

The man can recover damages for his injuries under the Consumer Protection Act 1987 because the bike is defective. The Act imposes strict liability on the producer of a defective product for any damage caused by the defect. A product is considered defective if it does not provide the safety that a person is entitled to expect. In this case, the specially developed metal frame of the mountain bike was intended to enhance its performance over rough terrain. However, the metal corrosion in some of the screws caused the handlebar to snap, which was unforeseeable. As a result, the bike did not provide the safety that the man was entitled to expect. Therefore, the man can recover damages for his injuries under the Consumer Protection Act 1987 because the bike is defective and the injuries were caused by the defect. The fact that the injuries were not foreseeable or that damages for personal injuries are not awarded under the Act is not relevant in this case.

Question 22[edit | edit source]

Legal Services:

https://sqe.sra.org.uk/exam-arrangements/assessment-information/sqe1-sample-questions/flk1/question22

E is the best answer.

A “success fee” is the added fee that a solicitor becomes entitled to under a CFA if their client's case is successful. Upon success the solicitor is entitled to standard hourly rates plus a success fee calculated as a % uplift on these hourly rates, to a maximum of 100%. Here, the claimant’s case was successful so the standard hourly rates of £20,000, added by £20,000 X 0.9 (90% success fee) equals to £38,000.

Question 23[edit | edit source]

Legal Services:

https://sqe.sra.org.uk/exam-arrangements/assessment-information/sqe1-sample-questions/flk1/question23

The correct answer is B.

The Court of Appeal is not bound to the earlier decision of the Judicial Committee of the Privy Council (JCPC). This was clarified by the Supreme Court in the case of Willers v Joyce & anr [2016] UKSC 43. In that case, the Supreme Court held that the Court of Appeal should not follow a decision of the JCPC if it is inconsistent with a decision that would otherwise be binding on the lower court, unless the JCPC has expressly directed that domestic courts should treat its decision as representing the law of England and Wales. The JCPC was the highest court of appeal for England and Wales until 2009. However, the Supreme Court is now the highest court in England and Wales. The Supreme Court is not bound by the decisions of the JCPC. This is because the Supreme Court is a superior court to the JCPC. The Supreme Court should only depart from the decisions of the JCPC if there is good reason to do so.

Question 24[edit | edit source]

Public Law: https://sqe.sra.org.uk/exam-arrangements/assessment-information/sqe1-sample-questions/flk1/question24

The best answer is A.

Under Article 3 of the Convention, “No one shall be subjected to torture or to inhuman or degrading treatment or punishment.” As a signatory to the Convention, the UK is obliged to secure the Convention rights for everyone within its jurisdiction, including non-UK nationals. Therefore, the High Court would not uphold the deportation order.

Question 25[edit | edit source]

Business Law and Practice:

https://sqe.sra.org.uk/exam-arrangements/assessment-information/sqe1-sample-questions/flk1/question25

The best answer is A.

A partnership is created when two or more people agree to carry on a business together with a view to making a profit. In this case, the three clients have agreed to go into business together and share the profits equally. Although the business has made a loss, the fact that the clients intended to make a profit and invested capital in the business indicates that they were working in partnership together.

Question 26[edit | edit source]

Ethics:

https://sqe.sra.org.uk/exam-arrangements/assessment-information/sqe1-sample-questions/flk1/question26

E is the best answer.

Under 8.7 SRA Code of Conduct for Solicitors, RELs and RFLs, “You ensure that clients receive the best possible information about how their matter will be priced and, both at the time of engagement and when appropriate as their matter progresses, about the likely overall cost of the matter and any costs incurred.” If the fee is going to exceed that which has been agreed, the solicitor must ensure that he receives the client's agreement to that fee increase before commencing work or else you risk not being paid. Note that costs means “fees and disbursements” under SRA glossary. A is wrong. The solicitor needs to inform the client regarding the cost as the matter progresses. Under 8.7 SRA Code of Conduct for Solicitors, RELs and RFLs. B is wrong. The solicitor needs to inform the client regarding the cost as the matter progresses. Under 8.7 SRA Code of Conduct for Solicitors, RELs and RFLs. C is wrong. The solicitor needs to inform the client regarding the cost as the matter progresses. Under 8.7 SRA Code of Conduct for Solicitors, RELs and RFLs. Note that costs means “fees and disbursements” where disbursements mean “any costs or expenses paid or to be paid to a third party on behalf of the client or trust (including any VAT element) save for office expenses such as postage and courier fees” D is wrong. That there is no general principle to the effect that a client must specifically approve in advance any disbursements incurred by a solicitor, if they are to be recoverable from the client, and recovery of costs and disbursements does not turn only upon whether they have been incurred with the client’s “informed consent”. Even if a given item of cost, or a given disbursement, does not fall within an estimate already given or has not been expressly authorised in advance, it simply does not follow that informed consent to the expenditure has not been given.

Question 27[edit | edit source]

COntract Law:

https://sqe.sra.org.uk/exam-arrangements/assessment-information/sqe1-sample-questions/flk1/question27

The best answer is C. When a buyer rejects goods, they are entitled to recover the purchase price paid, any expenses incurred in accepting and returning the goods, and any other losses directly resulting from the breach of contract. In this case, the Retailer rejected the defective boxes and obtained replacement boxes from another supplier at a cost of £70,000. The Retailer is entitled to recover the purchase price paid to the Supplier, which was £60,000, as well as the additional expense of obtaining replacement boxes, which was £70,000, but only to the extent that this additional expense was a loss directly resulting from the breach of contract. The deposit of £20,000 paid by the Retailer is not relevant to the amount of the claim, as it was paid towards the purchase price and is offset by the fact that the Retailer did not pay the balance of the purchase price. Therefore, the sum that the Retailer can properly claim against the Supplier is the difference between the cost of the replacement boxes and the purchase price paid to the Supplier, which is £70,000 - £60,000 = £10,000. In addition, the Retailer can claim any other losses directly resulting from the breach of contract, such as the cost of any additional transportation or storage costs incurred as a result of the defective boxes. However, the question only asks for the sum of the claim, so the correct answer is C. £30,000.

Question 28[edit | edit source]

Tort Law: https://sqe.sra.org.uk/exam-arrangements/assessment-information/sqe1-sample-questions/flk1/question28

A is the best answer.

An employer can be vicariously liable for wrongful acts committed by one of its employees where there is a sufficient connection between those wrongful acts and the employee’s employment. B is wrong. The Supreme Court holds that employers can be vicariously liable for the acts of their employees who commit a crime whilst at work. WM Morrison Supermarkets plc v Various Claimants C is wrong. Employers can be vicariously liable for incidents that happen out of hours. Bellman v Northampton Recruitment Ltd D is wrong. Employers may be vicariously liable for the actions of “all” their employees whether they are junior or senior. E is wrong. Vicarious liability arises even though the servant acted against the express instructions, and for no benefit of his master.

Question 29[edit | edit source]

Legal Services:

https://sqe.sra.org.uk/exam-arrangements/assessment-information/sqe1-sample-questions/flk1/question29

The best answer is B.

In this case, the solicitor has reasonable grounds to suspect that his client is involved in money laundering activities, which is a criminal offense. The Solicitors Regulation Authority (SRA) requires solicitors to report any suspicion of money laundering to the relevant authority and to take steps to prevent any further involvement in the transaction until they receive authorization from the relevant authority. The solicitor has reported the suspicious activity to the nominated officer, but that does not necessarily absolve them of their ethical and legal obligations. The solicitor must ensure that they comply with their professional obligations, including the duty to act with integrity and in the best interests of their client, while also complying with the legal and regulatory requirements in relation to money laundering.

Therefore, the solicitor should not proceed with the transaction until they receive authorization from the relevant authority, as failure to do so could potentially result in them being involved in criminal activity and being subject to disciplinary action by the SRA.

Question 30[edit | edit source]

Contract Law:

https://sqe.sra.org.uk/exam-arrangements/assessment-information/sqe1-sample-questions/flk1/question30

The best answer is D. Restitution refers to the restoration of the parties to their original position before the contract was made. In this case, restitution is impossible because the buyer has carried out extensive building works on the nursery after purchasing it. The buyer cannot return the nursery to the seller in its original condition. This means that rescission, which is based on the principle of restitution, is not an available remedy for the buyer. Instead, the buyer may be able to claim damages for the fraudulent misrepresentation made by the seller.

Question 31[edit | edit source]

Legal Services:

https://sqe.sra.org.uk/exam-arrangements/assessment-information/sqe1-sample-questions/flk1/question31

The best answer is B. The Equality Act 2010 requires employers to make reasonable adjustments for disabled employees to prevent them from being placed at a substantial disadvantage compared to non-disabled employees. This duty arises when a provision, criterion or practice, a physical feature or the absence of an auxiliary aid or service puts a disabled person at a disadvantage compared to non-disabled persons. In this case, the disabled employee has requested that the firm buy computer software that would enable him to use a computer more effectively. If the firm fails to provide this reasonable adjustment, the disabled employee may be placed at a substantial disadvantage compared to non-disabled employees who are able to use a computer without such software. This may lead to the disabled employee being excluded from certain job opportunities or unable to perform his role effectively, which could be considered discrimination under the Act. Therefore, to meet their obligations under the Act, the partners in the firm must make reasonable adjustments, such as providing the necessary computer software, to ensure that the disabled employee is not placed at a substantial disadvantage.

Question 32[edit | edit source]

Legal Services:

https://sqe.sra.org.uk/exam-arrangements/assessment-information/sqe1-sample-questions/flk1/question32

C is the best answer.

Firms must identify their customers and, where applicable, their beneficial owners, and then verify their identities. Under the Money Laundering, Terrorist Financing and Transfer of Funds (Information on the Payer) Regulations 2017, Meaning of beneficial owner: bodies corporate or partnership 5.—(1) In these Regulations, “beneficial owner”, in relation to a body corporate which is not a company whose securities are listed on a regulated market, means— (a) any individual who exercises ultimate control over the management of the body corporate; (b) any individual who ultimately owns or controls (in each case whether directly or indirectly), including through bearer share holdings or by other means, more than 25% of the shares or voting rights in the body corporate; or A, B, D, and E are wrong.

Question 33[edit | edit source]

Dispute Resolution:

https://sqe.sra.org.uk/exam-arrangements/assessment-information/sqe1-sample-questions/flk1/question33

The best answer is E.

The general rule in litigation is that the unsuccessful party pays the costs of the successful party, and in this case, the man was unsuccessful in defending the claim brought by the estate agent.

Question 34[edit | edit source]

Tort Law

https://sqe.sra.org.uk/exam-arrangements/assessment-information/sqe1-sample-questions/flk1/question34

A is the best answer.

In tort actions, failing to mitigate signifies that a party has not taken the necessary actions to prevent additional damage to its property. B is wrong. Contributory negligence is not a complete bar to damage. The Law Reform (Contributory Negligence) Act 1945 made it possible for judges to make awards of damages to plaintiffs who had been in part responsible, by reason of their own negligence, for causing their injury: prior to this Act, if the plaintiff were at all negligent no damages could be awarded. C is wrong. Rylands v Fletcher requires 1. Accumulation on the defendant's land 2. A thing likely to do mischief if it escapes 3. Escape 4. Non-natural use of land 5. The damage must not be too remote. In the factual scenario, the accumulation is on company’s or claimant’s land and not woman’s. D is wrong. The cost of the repair to the factory floor is not in the reasonable contemplation of the company because the company did move the drums even though it could have moved them at no cost. E is wrong. The chain of causation in the damage is not broken- the damage is caused by drums not being suitable for the storage of the acid and the company’s inaction (not moving the drums) did not break that chain.

Question 35[edit | edit source]

Ethics:

https://sqe.sra.org.uk/exam-arrangements/assessment-information/sqe1-sample-questions/flk1/question35

D is the best answer.

SRA Code of Conduct for Solicitors, RELs and RFLs 5.1(a) states regarding any referral of a client, “clients are informed of any financial or other interest which you or your business or employer has in referring the client to another person or which an introducer has in referring the client to you.”

Question 36[edit | edit source]

Public Law:

https://sqe.sra.org.uk/exam-arrangements/assessment-information/sqe1-sample-questions/flk1/question36

The best answer is E.

In judicial review, the test for apparent bias is objective: would “the fair-minded and informed observer, having considered the facts…conclude that there was a real possibility that the tribunal was biased”.

Question 37[edit | edit source]

Ethics

https://sqe.sra.org.uk/exam-arrangements/assessment-information/sqe1-sample-questions/flk1/question37 The correct answer is D.

Question 38[edit | edit source]

Tort Law

https://sqe.sra.org.uk/exam-arrangements/assessment-information/sqe1-sample-questions/flk1/question38

A is the best answer.

In contributory negligence, negligent action by the injured must have contributed to his own injury or loss. B is wrong. C is wrong. D is wrong. E is wrong.

Question 39[edit | edit source]

Legal Services

https://sqe.sra.org.uk/exam-arrangements/assessment-information/sqe1-sample-questions/flk1/question39

The best answer is C.

Generic advice does not constitute investment advice as defined in article 53 of the Article 53 of the Financial Services and Markets Act 2000 (Regulated Activities) Order 2001 as amended (RAO)).

Question 40[edit | edit source]

Public Law

https://sqe.sra.org.uk/exam-arrangements/assessment-information/sqe1-sample-questions/flk1/question40

The best answer is A.

The Parliament Acts have been used to pass legislation against the wishes of the House of Lords on seven occasions since 1911, including the passing of the Parliament Act 1949.

B, C, D and E are wrong.

Question 41[edit | edit source]

Tort Law:

https://sqe.sra.org.uk/exam-arrangements/assessment-information/sqe1-sample-questions/flk1/question41

D is the best answer.

In this scenario, the woman developed traumatic neurosis as a result of witnessing the man drowning due to the negligence of a qualified instructor during a swimming lesson. However, the instructor did not owe the woman a duty of care, and therefore, the woman cannot recover damages against the instructor for the traumatic neurosis. A duty of care is a legal obligation to take reasonable care to avoid causing harm to others. In order to establish a duty of care, it is necessary to show that there was a foreseeable risk of harm, that the defendant (in this case, the instructor) had a relationship of proximity or neighbourhood to the claimant (in this case, the woman), and that it is fair, just and reasonable to impose a duty of care in the circumstances. In this case, the instructor owed a duty of care to the man who was taking the swimming lesson. The instructor was responsible for ensuring that the man was properly supervised and safe in the pool. However, the instructor did not owe a duty of care to the woman who was a mere bystander at the pool. The instructor had no relationship of proximity or neighbourhood to the woman, and it was not foreseeable that the woman would suffer harm as a result of witnessing the man's drowning. Therefore, the instructor did not owe a duty of care to the woman, and the woman cannot recover damages against the instructor for the traumatic neurosis. The woman may be able to recover damages if she can establish that someone else owed her a duty of care, and that person's negligence caused her harm.

A is wrong. B is wrong. C is wrong. E is wrong.

Question 42[edit | edit source]

Dispute Resolution

https://sqe.sra.org.uk/exam-arrangements/assessment-information/sqe1-sample-questions/flk1/question42

The best answer is A.

CPR 1.1 No, these instructions are not likely to be consistent with the overriding objective of the Civil Procedure Rules because they do not reflect the requirement to deal with cases justly and at proportionate cost. The overriding objective of the Civil Procedure Rules is to ensure that cases are dealt with justly and at proportionate cost. This means that parties to a case should act in a way that helps the court to deal with the case efficiently, fairly, and at a proportionate cost. The CPR encourages parties to settle disputes wherever possible, and to use appropriate dispute resolution procedures to help achieve this objective. In this case, the PLC has instructed its solicitor to compile a list of documents containing 267 pages of emails and to seek expert evidence from two different experts on vending machines. This approach does not reflect the requirement of dealing with cases at proportionate cost. Seeking expert evidence and compiling such a large number of documents can be time-consuming and expensive, and may not be necessary to determine the issues in dispute. Moreover, the issue in dispute is a simple one: whether the school owes PLC the sum of £300 due under the lease agreement for the vending machine. Seeking expert evidence on vending machines may not be relevant or proportionate to the issue in dispute. Therefore, these instructions are not likely to be consistent with the overriding objective of the Civil Procedure Rules, which requires parties to help the court deal with cases justly and at proportionate cost. The parties should focus on identifying the relevant issues in dispute, and on using proportionate and cost-effective means to resolve them.

Question 43[edit | edit source]

Contract Law

https://sqe.sra.org.uk/exam-arrangements/assessment-information/sqe1-sample-questions/flk1/question43

B is the best answer.

In the rule in Pinnel’s case (1602) 5 Co Rep 117, part payment of a debt will not satisfy the obligation to repay the entire debt, except in a few circumstances or if drafted in a duly executed deed of settlement. This is because there is no fresh consideration provided for the second “agreement” and is therefore not binding on the parties. There is no fresh consideration because a promise to perform an existing legal obligation is not good consideration. A is wrong because duress occurs where someone enters into a contract as a result of undue pressure. Examples of duress are threats of violence, unlawful restraint, to property which do not apply to this factual scenario. The creditor may claim economic duress which requires (1) an illegitimate threat; (2) sufficient causation - in other words, that the illegitimate threat caused the threatened party to enter into the contract; and (3) that the threatened party had no reasonable alternative to giving in to the threat. In the factual scenario, there is no illegitimate threat or reasonable alternative so the duress argument would probably not succeed here. C is wrong because until the fresh new consideration is provided the claim to the full amount is not waived. D is wrong because the creditor is already entitled to £4,000 so there is no fresh new consideration (or legal benefit or detriment) for the second agreement for part debt. E is wrong because no practical benefit has been conferred to the creditor. If a person promises more than what they originally did under a contract, and they will get something more than what they were already legally entitled to under the contract, then they will receive a practical benefit and that additional offer will be binding on that person.

Question 44[edit | edit source]

Tort Law:

https://sqe.sra.org.uk/exam-arrangements/assessment-information/sqe1-sample-questions/flk1/question44

B is the best answer. Under the legal principle of 'thin skull rule' or 'eggshell skull rule', a defendant must take the plaintiff as they find them, meaning that a defendant is liable for all damages resulting from their negligence, even if the plaintiff's injuries are more severe than they would be for an ordinary person.

Question 45[edit | edit source]

Business Law and Practice:

https://sqe.sra.org.uk/exam-arrangements/assessment-information/sqe1-sample-questions/flk1/question45

The correct answer is D.

Under English tax law, dividends are taxed differently from other forms of income such as salary or wages. The first £2,000 of dividend income in the tax year is tax-free, but any amount over this is subject to taxation at the appropriate dividend tax rate. In this case, the woman received a dividend of £5,000 in the 2021/22 tax year, which is £3,000 over the dividend allowance of £2,000. Therefore, she should pay Income Tax at the appropriate dividend tax rate on this £3,000 excess. Her total taxable income for the year, including the dividend, is £41,000, which is within the basic rate tax band of £12,571-£50,270. The basic rate of Income Tax for the 2021/22 tax year is 20%. Therefore, the woman should pay tax on the £3,000 excess of her dividend income at the basic rate of 20%, which amounts to £600. She should report this dividend income on her Self-Assessment tax return and pay any tax due by the deadline of January 31st following the end of the tax year.

Question 46[edit | edit source]

Legal Services:

https://sqe.sra.org.uk/exam-arrangements/assessment-information/sqe1-sample-questions/flk1/question46

The best answer is A.

Question 47[edit | edit source]

Contract Law: https://sqe.sra.org.uk/exam-arrangements/assessment-information/sqe1-sample-questions/flk1/question47

The best answer is A.

Question 48[edit | edit source]

Business Law and Practice: Business organisations, rules and procedures; Insolvency (corporate and personal): claw-back of assets for creditors – preferences, transactions at an undervalue, fraudulent and wrongful trading, setting aside a floating charge

https://sqe.sra.org.uk/exam-arrangements/assessment-information/sqe1-sample-questions/flk1/question48

The best answer is C.

Question 49[edit | edit source]

The Legal System: Statutory interpretation: mischief rule https://sqe.sra.org.uk/exam-arrangements/assessment-information/sqe1-sample-questions/flk1/question49

The best answer is A.

Question 50[edit | edit source]

Tort:

https://sqe.sra.org.uk/exam-arrangements/assessment-information/sqe1-sample-questions/flk1/question50

The best answer is D.

Question 51[edit | edit source]

Dispute Resolution:

https://sqe.sra.org.uk/exam-arrangements/assessment-information/sqe1-sample-questions/flk1/question51

The best answer is E.

Question 52[edit | edit source]

Legal Services:

https://sqe.sra.org.uk/exam-arrangements/assessment-information/sqe1-sample-questions/flk1/question52

The best answer is A.

Question 53[edit | edit source]

Business Law:

https://sqe.sra.org.uk/exam-arrangements/assessment-information/sqe1-sample-questions/flk1/question53

The best answer is C.

Question 54[edit | edit source]

Taxation:

https://sqe.sra.org.uk/exam-arrangements/assessment-information/sqe1-sample-questions/flk1/question54

The best answer is A.

Question 55[edit | edit source]

Contract Law: https://sqe.sra.org.uk/exam-arrangements/assessment-information/sqe1-sample-questions/flk1/question55

The best answer is C.

Question 56[edit | edit source]

Public Law: Human Rights

https://sqe.sra.org.uk/exam-arrangements/assessment-information/sqe1-sample-questions/flk1/question56

The best answer is A.

Question 57[edit | edit source]

Legal System:

https://sqe.sra.org.uk/exam-arrangements/assessment-information/sqe1-sample-questions/flk1/question57

The best answer is B.

Question 58[edit | edit source]

Tort Law:

https://sqe.sra.org.uk/exam-arrangements/assessment-information/sqe1-sample-questions/flk1/question58

The best answer is A.

Question 59[edit | edit source]

Legal Services:

https://sqe.sra.org.uk/exam-arrangements/assessment-information/sqe1-sample-questions/flk1/question59

The best answer is C.

Question 60[edit | edit source]

Taxation

https://sqe.sra.org.uk/exam-arrangements/assessment-information/sqe1-sample-questions/flk1/question60

The best answer is E.

Question 61[edit | edit source]

Contract Law:

https://sqe.sra.org.uk/exam-arrangements/assessment-information/sqe1-sample-questions/flk1/question61

The best answer is D.

Question 62[edit | edit source]

Dispute Resolution:

https://sqe.sra.org.uk/exam-arrangements/assessment-information/sqe1-sample-questions/flk1/question62

The best answer is A.

Question 63[edit | edit source]

Public Law:

https://sqe.sra.org.uk/exam-arrangements/assessment-information/sqe1-sample-questions/flk1/question63

The best answer is E.

Question 64[edit | edit source]

Legal Services https://sqe.sra.org.uk/exam-arrangements/assessment-information/sqe1-sample-questions/flk1/question64

The best answer is A.

Question 65[edit | edit source]

Tort Law

https://sqe.sra.org.uk/exam-arrangements/assessment-information/sqe1-sample-questions/flk1/question65

The best answer is B.

FLK2 Answers and Explanations[edit | edit source]

Question 1[edit | edit source]

Criminal Procedure:

https://sqe.sra.org.uk/exam-arrangements/assessment-information/sqe1-sample-questions/flk2/question1

The best answer is B.

Under the PACE 1984 (Codes of Practice) (Temporary Modifications to Code D) Order 2002, Circumstances in which an identification procedure must be held

2.15 An identification procedure need not be held if, in all the circumstances, it would serve no useful purpose in proving or disproving whether the suspect was involved in committing the offence. Examples would be where it is not in dispute that the suspect is already well known to the witness who saw the suspect commit the crime or where there is no reasonable possibility that a witness would be able to make an identification. Here the suspect is already known to the witness so the identification procedure would not serve a useful purpose in this case.

Question 2[edit | edit source]

Criminal Procedure:

https://sqe.sra.org.uk/exam-arrangements/assessment-information/sqe1-sample-questions/flk2/question2

The best answer is A.

The 'operational period' of the suspended sentence is the period between 6 months and 2 years. If there is a conviction for further offence committed during operational period of order, the court must activate the custodial sentence unless it would be unjust in all the circumstances to do so.

Question 3[edit | edit source]

Trusts Law:

https://sqe.sra.org.uk/exam-arrangements/assessment-information/sqe1-sample-questions/flk2/question3

The best answer is E. Under Trustee Act 1925, Devolution of powers or trusts.

(1) Where a power or trust is given to or imposed on two or more trustees jointly, the same may be exercised or performed by the survivors or survivor of them for the time being. (2) Until the appointment of new trustees, the personal representatives or representative for the time being of a sole trustee, or, where there were two or more trustees of the last surviving or continuing trustee, shall be capable of exercising or performing any power or trust which was given to, or capable of being exercised by, the sole or last surviving or continuing trustee, or other the trustees or trustee for the time being of the trust.

Question 4[edit | edit source]

Wills: https://sqe.sra.org.uk/exam-arrangements/assessment-information/sqe1-sample-questions/flk2/question4

The best answer is B.

The executor should place the advertisements to notify any potential creditors who may have a claim against the deceased's estate. Placing such ads is a legal requirement to protect the executor from future claims by creditors or beneficiaries who were not aware of the deceased's death or the distribution of the estate. By advertising in the London Gazette and other publications, the executor is providing a public notice to any potential creditors to come forward and make a claim against the estate. This helps ensure that the estate's assets are distributed to the rightful beneficiaries, and any legitimate claims by creditors are settled. The reason for waiting for two months from the date of the advertisements is to provide sufficient time for any potential creditors to come forward and make a claim against the estate. Once the two-month period has elapsed, the executor can distribute the estate among the beneficiaries with the assurance that any legitimate claims against the estate have been settled or accounted for. If the executor distributes the estate before the two-month period elapses and a creditor comes forward with a legitimate claim, the executor may be personally liable for any debts or obligations owed by the deceased that were not accounted for in the distribution. By waiting for the two-month period, the executor can protect themselves against such claims and ensure that the estate is distributed fairly and according to the law.

Question 5[edit | edit source]

Wills https://sqe.sra.org.uk/exam-arrangements/assessment-information/sqe1-sample-questions/flk2/question5 The correct answer is D. In this scenario, the testator appointed his spouse, his friend, and his adult son as executors in his will. However, as the testator and his spouse got divorced after the will was executed, the spouse is no longer eligible to be an executor.

Furthermore, the testator's son predeceased him, and a grant of probate to the son's estate was obtained by his nephew. Therefore, the son is no longer eligible to act as an executor.

This leaves the testator's friend as the only surviving appointed executor, and he has the best right to apply for a grant of representation to the testator's estate. The friend can apply for a grant of probate as the sole surviving executor and carry out the duties prescribed in the will.

Question 6[edit | edit source]

Ethics https://sqe.sra.org.uk/exam-arrangements/assessment-information/sqe1-sample-questions/flk2/question6

The correct answer is E.

The solicitor has a duty to act in the best interests of their client, but they also have a duty to uphold the law and the administration of justice. In this scenario, the client has provided false information to the court by giving a false address and date of birth to conceal previous convictions. This conduct undermines the administration of justice and violates the solicitor's duty to uphold the law. Therefore, the solicitor should ask the client to correct the information about her address and date of birth. The solicitor should explain to the client that providing false information to the court is illegal and could lead to serious consequences, including potential criminal charges for perjury or contempt of court. If the client refuses to correct the false information, the solicitor should cease to act for the client. Continuing to act for the client in such circumstances would be a breach of the solicitor's professional obligations and could expose the solicitor to disciplinary action, legal liability, and reputational damage. In addition, by ceasing to act, the solicitor would be preventing further unethical or illegal conduct by the client. The solicitor can advise the client to seek alternative legal representation and can take steps to withdraw from the case in a manner consistent with the professional rules of conduct.

Overall, the solicitor must balance their duties to the client with their duty to uphold the law and the administration of justice. In this scenario, asking the client to correct the false information is the appropriate course of action, but ceasing to act is necessary if the client refuses to comply.

Question 7[edit | edit source]

Land Law https://sqe.sra.org.uk/exam-arrangements/assessment-information/sqe1-sample-questions/flk2/question7

The best answer is B.

However, the freeholder obtained authorised guarantee agreements from the newsagent and the chemist. An authorised guarantee agreement is a legal document in which the assignor (the newsagent or the chemist) agrees to guarantee the assignee's (the clothing retailer or the bookstore) performance of the lease obligations. Therefore, in the event of a breach of the lease by the assignee, the freeholder can pursue the assignor for any losses incurred. In this scenario, as the freeholder required an authorised guarantee agreement from the chemist when the lease was assigned to the bookstore, the chemist is liable to the freeholder for any unpaid rent by the bookstore. The newsagent and the clothing retailer are not liable as they are not parties to the current lease, and the freeholder did not obtain an authorised guarantee agreement from the clothing retailer.

Question 8[edit | edit source]

FL2 - Land Law https://sqe.sra.org.uk/exam-arrangements/assessment-information/sqe1-sample-questions/flk2/question8

The best answer is D.

Under 26, Tenant’s request for a new tenancy. (1) A tenant’s request for a new tenancy may be made where the [F51current tenancy] is a tenancy granted for a term of years certain exceeding one year, whether or not continued by section twenty-four of this Act, or granted for a term of years certain and thereafter from year to year. (2) A tenant’s request for a new tenancy shall be for a tenancy beginning with such date, not more than twelve nor less than six months after the making of the request, as may be specified therein: Provided that the said date shall not be earlier than the date on which apart from this Act the current tenancy would come to an end by effluxion of time or could be brought to an end by notice to quit given by the tenant. (3) A tenant’s request for a new tenancy shall not have effect unless it is made by notice in the prescribed form given to the landlord and sets out the tenant’s proposals as to the property to be comprised in the new tenancy (being either the whole or part of the property comprised in the current tenancy), as to the rent to be payable under the new tenancy and as to the other terms of the new tenancy. (4) A tenant’s request for a new tenancy shall not be made if the landlord has already given notice under the last foregoing section to terminate the current tenancy, or if the tenant has already given notice to quit or notice under the next following section; and no such notice shall be given by the landlord or the tenant after the making by the tenant of a request for a new tenancy. (5) Where the tenant makes a request for a new tenancy in accordance with the foregoing provisions of this section, the current tenancy shall, subject to the provisions of [F52sections 29B(4) and 36(2)] of this Act and the provisions of Part IV of this Act as to the interim continuation of tenancies, terminate immediately before the date specified in the request for the beginning of the new tenancy. (6) Within two months of the making of a tenant’s request for a new tenancy the landlord may give notice to the tenant that he will oppose an application to the court for the grant of a new tenancy, and any such notice shall state on which of the grounds mentioned in section thirty of this Act the landlord will oppose the application.

Under Landlord and Tenant Act 1954 33. Duration of new tenancy- Where on an application under this Part of this Act the court makes an order for the grant of a new tenancy, the new tenancy shall be such tenancy as may be agreed between the landlord and the tenant, or, in default of such an agreement, shall be such a tenancy as may be determined by the court to be reasonable in all the circumstances, being, if it is a tenancy for a term of years certain, a tenancy for a term not exceeding [fifteen] years, and shall begin on the coming to an end of the current tenancy.

Question 9[edit | edit source]

Trust Law https://sqe.sra.org.uk/exam-arrangements/assessment-information/sqe1-sample-questions/flk2/question9

The correct answer is E.

This is a fixed trust and certainty of objects is determined because the trustees can compile a complete list of all beneficiaries from the complete record of past and present employees of the company.

Question 10[edit | edit source]

Wills https://sqe.sra.org.uk/exam-arrangements/assessment-information/sqe1-sample-questions/flk2/question10

The correct answer is A.

The mother’s interested vested whereas those for the nephew and niece are not because there are two conditions, (1) to survive her and attain the age of 21, have not occurred yet. Therefore, the interests for the nephew and niece are contingent.

Question 11[edit | edit source]

Property Practice https://sqe.sra.org.uk/exam-arrangements/assessment-information/sqe1-sample-questions/flk2/question11

The answer is D.

Registration of the dealing at the Land Registry creates a legal record of the transfer of ownership from the seller to the purchaser. Once the registration is complete, the purchaser's title is protected from any subsequent entries or claims made against the property. However, until the registration is complete, the title is vulnerable to other interests and charges being entered against it, which could potentially bind the purchaser. The 30 working days deadline is important because it ensures that the registration is completed in a timely manner and the purchaser's title is protected as soon as possible after completion. If the registration is not completed within this time frame, there is a risk that other interests or charges could be entered against the title, which could affect the purchaser's ownership of the property.

Question 12[edit | edit source]

Land Law https://sqe.sra.org.uk/exam-arrangements/assessment-information/sqe1-sample-questions/flk2/question12

The best answer is D.

Fixtures are chattels, which have become apart of the realty by virtue of having been attached to it in a particular way, or annexed to it. The doctrine of fixtures provides that personal property may become real property if it is annexed or attached to land. Degree of Annexation: Objective test determined at the time the object was annexed to the land Factors: o Strength of attachment (NAB) o Easy removal (NAB = not heavy and easy to remove) o Damage and cost upon removal – economic consequences flowing from removal (NAB) ▪ Would it cost more to remove and reinstall than the article’s worth (super heavy?)

Question 13[edit | edit source]

FLK2- Property Practice https://sqe.sra.org.uk/exam-arrangements/assessment-information/sqe1-sample-questions/flk2/question13

The best answer is A.

Other than in defined circumstances, any change of use to or from such uses requires full local consideration through a planning application process.

Question 14[edit | edit source]

Criminal Procedure https://sqe.sra.org.uk/exam-arrangements/assessment-information/sqe1-sample-questions/flk2/question14

The best answer is D.

The man is in breach of his bail condition not to contact his girlfriend by visiting her at her flat. Even though the visit was at the request of his girlfriend, it still amounts to contact and therefore a breach of the condition. The fact that the purpose of the visit was to attend to their sick child does not excuse the breach of the condition. The man may be brought before the court for his bail to be reconsidered in light of the alleged breach of the condition.

Question 15[edit | edit source]

FLK2- Wills https://sqe.sra.org.uk/exam-arrangements/assessment-information/sqe1-sample-questions/flk2/question15

The best answer is C.

As the man died intestate, his estate will be distributed in accordance with the rules of intestacy. Since the man was not married or in a civil partnership, his estate will be distributed among his children in equal shares, as set out in the Intestacy Rules. The partner is not entitled to the estate because the partner is neither married or in a civil partnership. Only the man’s son and daughter are entitled.

Question 16[edit | edit source]

FLK2- Criminal Practice https://sqe.sra.org.uk/exam-arrangements/assessment-information/sqe1-sample-questions/flk2/question16

C is the best answer.

Code D of PACE 1984 provides guidance on the identification of suspects by witnesses, including the use of video identification parades. One of the requirements of Code D is that the images of the suspects used in the video identification parade must be fairly similar to the suspect in question. In this case, the solicitor has identified that only two of the other eight images shown in the video parade show similar-looking males with a visible scar under their right eye. This could be seen as a provable breach of Code D, as the man's image may have been unfairly singled out for the identification parade. If the court agrees that there has been a breach of Code D, it may decide to exclude the identification evidence obtained from the video parade. The court has the power to exclude evidence if its admission would have an adverse effect on the fairness of the proceedings. This means that if the court finds that the breach of Code D has compromised the reliability of the identification evidence, its admission may be unfair to the defendant and therefore it should be excluded.


Question 17[edit | edit source]

Criminal Law https://sqe.sra.org.uk/exam-arrangements/assessment-information/sqe1-sample-questions/flk2/question17

The correct answer is E. For factual causation, D’s act does not have to be the only cause of the prohibited result, or even the main cause. It just has to be an ‘operating and substantial’ cause of the result

Question 18[edit | edit source]

FLK2- Criminal Law https://sqe.sra.org.uk/exam-arrangements/assessment-information/sqe1-sample-questions/flk2/question18

The correct answer is D.

The man's act of setting fire to the caravan was an unlawful act, and it was also dangerous because he did not check if anyone was inside before setting fire to it. This means that the man was engaged in an unlawful and dangerous act that put the lives of others at risk.

The death of the elderly man was a direct result of the man's actions. The fact that the man did not intend to harm anyone is irrelevant, as he still committed an unlawful and dangerous act that resulted in the death of another person. Therefore, the man can be charged with involuntary manslaughter, which is the unlawful killing of another person without the intent to cause death or serious harm, but as a result of a dangerous or unlawful act.

Question 19[edit | edit source]

Land Law https://sqe.sra.org.uk/exam-arrangements/assessment-information/sqe1-sample-questions/flk2/question19

The correct answer is D.

When the husband, wife, and wife's mother bought the house together, they did so as joint tenants in law and equity. This means that they all jointly own the property and each person has an equal share in the property. As joint tenants, they have a right of survivorship, which means that when one joint tenant dies, their share automatically passes to the surviving joint tenant(s) and not according to any will they may have made.

Therefore, when the wife died, her share in the property passed automatically to the remaining joint tenants, her mother and her husband. The will that the wife made leaving all her property to her new boyfriend is ineffective as it only applies to her personal property, not to property held jointly with others as joint tenants.

Question 20[edit | edit source]

Wills https://sqe.sra.org.uk/exam-arrangements/assessment-information/sqe1-sample-questions/flk2/question20

The best answer is A.

As personal representatives (PRs), the individuals responsible for administering the estate, they have a legal duty to take reasonable steps to identify and locate all potential creditors and beneficiaries. This includes making reasonable efforts to locate the man's son, as he is named as a beneficiary in the will.

By placing advertisements complying with s.27 Trustee Act 1925, the PRs have taken steps to notify any potential creditors and beneficiaries of the estate. However, these efforts may not be sufficient to discharge their duty to locate the son. If the son were to come forward at a later date and claim that he was not notified or made aware of the distribution, he may be able to challenge the distribution of the estate and claim his share.

In addition, if there were any unknown creditors of the estate, they may also have a claim against the PRs if the PRs did not take reasonable steps to identify and locate them. However, if the PRs made reasonable efforts to locate creditors and no claims were made within the relevant time limit, the PRs may be protected from future claims by unknown creditors.

Therefore, while the PRs have taken some steps to locate potential beneficiaries and creditors, they may not be fully protected from claims by the man's son.

Question 21[edit | edit source]

Property Practice https://sqe.sra.org.uk/exam-arrangements/assessment-information/sqe1-sample-questions/flk2/question21

The best answer is A.

When a person dies and their estate is subject to Inheritance Tax (IHT), the value of the assets in their estate must be determined. In this case, the sister’s estate includes her half share of the leasehold flat, which is valued at £100,000. However, because the flat was co-owned by the brother and sister as tenants in common, the value of the sister’s half share is subject to a percentage discount to reflect the fact that the brother already owns the other half of the flat. This is known as a 'fractional interest' discount. The exact percentage discount will depend on the circumstances, but it is typically around 10-15% for co-owned property. In this case, assuming a 10% discount, the value of the sister’s half share for IHT purposes would be £90,000 (£100,000 minus 10%). Therefore, the sister’s estate would be subject to IHT based on a total value of £90,000 for her half share of the leasehold flat, rather than the full market value of £100,000.

Question 22[edit | edit source]

Property Practice https://sqe.sra.org.uk/exam-arrangements/assessment-information/sqe1-sample-questions/flk2/question22

The best answer is A.

When a person dies and their estate is subject to Inheritance Tax (IHT), the value of the assets in their estate must be determined. In this case, the sister’s estate includes her half share of the leasehold flat, which is valued at £100,000. However, because the flat was co-owned by the brother and sister as tenants in common, the value of the sister’s half share is subject to a percentage discount to reflect the fact that the brother already owns the other half of the flat. This is known as a 'fractional interest' discount.

The exact percentage discount will depend on the circumstances, but it is typically around 10-15% for co-owned property. In this case, assuming a 10% discount, the value of the sister’s half share for IHT purposes would be £90,000 (£100,000 minus 10%).

Therefore, the sister’s estate would be subject to IHT based on a total value of £90,000 for her half share of the leasehold flat, rather than the full market value of £100,000.

Question 23[edit | edit source]

Criminal Law https://sqe.sra.org.uk/exam-arrangements/assessment-information/sqe1-sample-questions/flk2/question23

The correct answer is D.

The man is guilty of criminal damage because he intentionally squeezed superglue into the hard drive of his colleague's computer, which resulted in the computer being damaged beyond repair. While he did not intend to cause such damage, he foresaw the risk of damage and it was unreasonable for him to take that risk in the circumstances known to him.

Under the Criminal Damage Act 1971, a person commits an offense of criminal damage if they intentionally or recklessly destroy or damage property belonging to another. In this case, the man intentionally damaged his colleague's computer, and even though he did not intend to cause irreparable damage, his actions were reckless because he foresaw the risk of damage and it was unreasonable to take that risk. Therefore, he can be found guilty of criminal damage.

Question 24[edit | edit source]

Land Law https://sqe.sra.org.uk/exam-arrangements/assessment-information/sqe1-sample-questions/flk2/question24

The best answer is E.

The client is likely to be able to sue the neighbour under the covenant because the neighbour was the original party to the deed. The deed containing the covenant is a legally binding contract between the two parties, and the neighbour agreed not to let the roof fall into disrepair for the benefit of the client's house. This means that the neighbour has a legal obligation to maintain the roof and ensure that it does not fall into disrepair, and if they breach this obligation, the client can take legal action against them for breach of contract. As the covenant was given directly to the client's house, it is likely that the client has the legal standing to bring a claim for breach of the covenant.

Question 25[edit | edit source]

Wills https://sqe.sra.org.uk/exam-arrangements/assessment-information/sqe1-sample-questions/flk2/question25 The correct answer is A. The Capital Gains Tax (CGT) on the shares sold during the administration period will depend on the value of the shares at the date of death of the testator, also known as the probate value.

When the estate was distributed, the son received shares worth £75,000 (£150,000/2), which he later sold for £85,000, resulting in a gain of £10,000. As the son has made a gain, he will be liable to pay CGT on the amount of the gain.

On the other hand, the daughter received shares worth £75,000 (£150,000/2), which she later sold for £45,000, resulting in a loss of £30,000. As the daughter has made a loss, she will not be liable to pay CGT, but the loss may be used to offset gains made on other investments in the same tax year.

The executor will make neither a loss nor a gain regarding the CGT position on the shares sold during the administration period as the CGT liability will be borne by the beneficiaries who made gains on the sale of the shares.

Question 26[edit | edit source]

Criminal Practice https://sqe.sra.org.uk/exam-arrangements/assessment-information/sqe1-sample-questions/flk2/question26

The best answer is D.

An admission made by a defendant to someone else, such as a friend, can be admissible as an exception to the hearsay rule. This is because admissions are considered reliable evidence against the person who made them. The fact that the man admitted to killing his wife to his friend in the public house can be presented as evidence against him in court, even if he denies it during his interview with the police.

Question 27[edit | edit source]

Solicitors Account https://sqe.sra.org.uk/exam-arrangements/assessment-information/sqe1-sample-questions/flk2/question27

The best answer is D.

Question 28[edit | edit source]

Ethics https://sqe.sra.org.uk/exam-arrangements/assessment-information/sqe1-sample-questions/flk2/question28

The correct answer is C.

According to the SRA Code of Conduct for Solicitors, RELs and RFLs, a solicitor can represent a client at the police station if they have instructions from the client or an authorised third party. In this case, the woman is seeking to give instructions on behalf of her father and the solicitor can accept these instructions provided he is satisfied that the woman has the authority to give instructions.

Regarding the conflict of interest issue, the solicitor must carry out the necessary checks to ensure that there is no conflict or risk of conflict. In this case, the solicitor has previously represented the woman’s brother who was arrested for the same offence. However, the fact that the brother has already been charged with the offence means that there is no risk of conflict as the interests of the brother and the father are now separate. Therefore, the solicitor can represent the father at the police station.

Question 29[edit | edit source]

Trusts Law https://sqe.sra.org.uk/exam-arrangements/assessment-information/sqe1-sample-questions/flk2/question29

The correct answer is B.

The trust was created by the woman in her will, and the terms of the trust provide for the payment of income from the residuary estate to the husband for the remainder of his life, with the remainder of the estate passing to the son if he survives the husband and attains the age of 25 years, but to the sister absolutely if the son does not survive the husband. The trust is therefore a valid and subsisting trust, and can only be brought to an end with the consent of all parties with an interest in the trust.

In this case, the husband has a life interest in the trust, and therefore has a vested interest in the income from the trust. The son has a contingent interest in the trust, which will only vest if he survives the husband and attains the age of 25 years. The sister also has a contingent interest in the trust, which will only vest if the son does not survive the husband.

Therefore, in order to bring the trust to an end now, the consent of all parties with an interest in the trust is required. This includes the husband, the son, and the sister. If any of these parties do not consent to the trust being brought to an end, the trust will continue in accordance with its terms.

Question 30[edit | edit source]

Trusts Law https://sqe.sra.org.uk/exam-arrangements/assessment-information/sqe1-sample-questions/flk2/question30

The correct answer is C.

The trustees have a duty to act in the best interests of all beneficiaries of the trust. In this case, the son is entitled to his share of the estate since he has attained the age of 21, but the daughters are not yet entitled to receive their shares. Since the will does not extend the trustees' powers of advancement, they can only make an advancement to the elder daughter if it is in the interests of all the beneficiaries.

In deciding whether to make an advancement, the trustees must consider the needs of all beneficiaries and the purpose for which the money is required. They must also ensure that any advancement is not excessive and does not unduly prejudice the interests of the other beneficiaries.

In this case, the maximum amount that the trustees can apply towards the elder daughter's business is £20,000, which is one-third of the total estate. This is because the estate is to be divided equally between the three children, and the elder daughter is entitled to one-third of the estate when she attains the age of 21. However, any advancement should be made in such a way as to ensure that the younger daughter's interests are not prejudiced, and that she will still receive her share of the estate when she attains the age of 21.

Question 31[edit | edit source]

Criminal Law https://sqe.sra.org.uk/exam-arrangements/assessment-information/sqe1-sample-questions/flk2/question31

The correct answer is D.

The man is guilty of robbery because he has committed the act of theft by taking the laptop and at the same time used force or threatened to use force against the daughter, causing the woman to fear that the force would be used, in order to commit the theft. Under section 8(1) of the Theft Act 1968, force used immediately before or at the time of stealing, or in order to steal, is considered robbery. In this case, the man threatened to break the girl's neck, which clearly constitutes the use of force, and caused the woman to fear that he would use the force, and therefore meets the elements of robbery.

Question 32[edit | edit source]

Property Practice https://sqe.sra.org.uk/exam-arrangements/assessment-information/sqe1-sample-questions/flk2/question32

The correct answer is D.

The option to tax exercised by the freehold owner means that the lease of the commercial building is treated as a standard-rated supply for VAT purposes. As the insurance company only makes exempt supplies in the course of its business, it is not registered for VAT and is unable to recover VAT on any expenses incurred in making those supplies. This means that the VAT charged on the rent payable under the lease will be irrecoverable by the insurance company, resulting in a cost to the company. Therefore, the rent payable will be subject to VAT at 20%, which will be irrecoverable by the insurance company.

Question 33[edit | edit source]

Criminal Law https://sqe.sra.org.uk/exam-arrangements/assessment-information/sqe1-sample-questions/flk2/question33

The correct answer is A.

To prove the offence of wounding with intent to cause grievous bodily harm, the prosecution must demonstrate that the defendant had the necessary intention to cause serious harm at the time of the attack. In this case, the man's voluntary intoxication may have affected his ability to form the required intention.

If the man was so intoxicated that he did not have the necessary intention to cause serious harm, he cannot be convicted of the offence. However, the burden of proof is on the defence to establish that the man's intoxication was so severe that he lacked the required intent.

If the defence can prove that the man was so intoxicated that he did not have the necessary intention to cause serious harm, the prosecution may attempt to charge him with an alternative offence such as reckless or negligent wounding, which may carry a lesser sentence.

Question 34[edit | edit source]

Land Law https://sqe.sra.org.uk/exam-arrangements/assessment-information/sqe1-sample-questions/flk2/question34

The correct answer is C.

The sublease was granted with the consent of the landlord, which implies that the landlord recognized the subtenant as a tenant of the headlease. When the tenant surrendered the headlease to the landlord, the sublease automatically became the direct lease between the subtenant and the landlord. This is because the sublease was granted for a term that was shorter than the term of the headlease and was therefore valid at the time of the surrender. Thus, the subtenant becomes the immediate tenant of the landlord on the terms of the sublease.

Question 35[edit | edit source]

Criminal Law https://sqe.sra.org.uk/exam-arrangements/assessment-information/sqe1-sample-questions/flk2/question1

The best answer is C.

The group is guilty of burglary under s.9(1)(a) of the Theft Act 1968. To establish burglary, the prosecution must prove that the defendant entered a building or part of a building as a trespasser with the intent to steal, inflict grievous bodily harm or do unlawful damage. In this scenario, the group entered the office outside of working hours without permission or lawful authority, and therefore, they entered as trespassers. Moreover, once inside the office, they searched through drawers and found alcohol which they consumed without permission, and also stole £300. These actions show that the group had the necessary intent to steal when they entered the office as trespassers. Therefore, they are all guilty of burglary.

Question 36[edit | edit source]

FLK2- Property Practice https://sqe.sra.org.uk/exam-arrangements/assessment-information/sqe1-sample-questions/flk2/question36

The correct answer is A.

When examining the Land Registry official copy of the title to the property, the buyer's solicitor will look for an entry in the Property Register that refers to an easement. An easement is a legal right to use someone else's land for a specific purpose, such as accessing a road.

If the property benefits from a legal right of way on foot, across a neighbour's registered freehold property, then this right should be registered as an easement in the Property Register. The buyer's solicitor will look for the details of the easement, such as its location, extent, and any limitations or conditions attached to it.

If the easement is properly registered, then the buyer can be confident that they have a legal right to use the right of way to access the road. However, if the easement is not registered or there are discrepancies in the details provided, the buyer's solicitor may need to investigate further or raise queries with the seller's solicitor to ensure that the right of way is properly documented and registered.

Question 37[edit | edit source]

FLK2- Land Law https://sqe.sra.org.uk/exam-arrangements/assessment-information/sqe1-sample-questions/flk2/question37

The correct answer is C.

Under the terms of the lease, the Tenant must obtain the prior written consent of the Landlord before assigning or underletting the Property. The Landlord has the right to withhold consent, but must do so reasonably and not arbitrarily.

In this case, the Landlord is concerned that the proposed assignee, a highly profitable national supermarket chain, will compete with its own business, which operates a supermarket opposite the property. This is a valid concern for the Landlord, as it could potentially result in a loss of trade and revenue for their business.

Under these circumstances, the Landlord can lawfully withhold consent to the assignment on the basis of the assignee's business, as competition with the Landlord's own business is a ground for reasonably withholding consent. This is because the covenant against assignment in the lease is intended to protect the Landlord's interests, including their commercial interests.

However, it is important to note that the Landlord must act reasonably in withholding consent, and cannot do so arbitrarily or unreasonably. The Landlord should consider all the relevant factors, including the financial position of the Tenant, the nature and reputation of the proposed assignee, and the potential impact on the Landlord's business, before making a decision on whether or not to grant consent to the assignment.

Question 38[edit | edit source]

FLK2- Property Practice https://sqe.sra.org.uk/exam-arrangements/assessment-information/sqe1-sample-questions/flk2/question38

The correct answer is A.

A root of title is a document that establishes the ownership of a property and provides an unbroken chain of title from the present owner back to a point in time where the title is free from defects. When deducing title to a property, it is important to identify a good root of title as it is the starting point for establishing the title's validity.

In this case, the conveyance of the property, dated 10 March 1984, is a good root of title when deducing title to the property. This is because a conveyance is a legal document that transfers the ownership of the property from the seller to the buyer, and it is considered a primary source of evidence of ownership.

Furthermore, the conveyance is dated more than 15 years ago, which means that it is outside the limitation period for challenging the title based on defects or errors in the conveyance. This means that the title based on the conveyance is likely to be considered good, and any subsequent conveyances or documents can be traced back to it as the root of title.

Therefore, the conveyance of the property dated 10 March 1984 is a good root of title when deducing title to the property, as it establishes the initial ownership of the property and provides an unbroken chain of title from that point in time.

Question 39[edit | edit source]

FLK1/2- Ethics https://sqe.sra.org.uk/exam-arrangements/assessment-information/sqe1-sample-questions/flk2/question39

The correct answer is E.

As per the SRA Standards and Regulations, a solicitor must act in the best interests of their client and avoid any conflicts of interest that may arise in the course of their professional duties. Specifically, a solicitor cannot act if there is an own interest conflict or a significant risk of an own interest conflict.

In this case, the solicitor was acting as both the seller and the buyer's solicitor in the sale of the buy-to-let flat. This creates an own interest conflict, as the solicitor has a personal interest in the transaction as the seller, which may conflict with the interests of their client, the buyer. Moreover, the client was also a pre-existing client of the solicitor. By accepting the instruction to act as the buyer's solicitor in the transaction, the solicitor created a conflict of interest between their professional duty to act in the best interests of their client and their personal interest in the sale of the property.

Therefore, the solicitor's action was not in accordance with the SRA Standards and Regulations, as they failed to avoid an own interest conflict and a significant risk of an own interest conflict by acting as both the seller and the buyer's solicitor in the transaction. This is because the solicitor's personal interest in the sale of the property could have conflicted with their duty to act in the best interests of their client.

Question 40[edit | edit source]

FLK2- Criminal Practice https://sqe.sra.org.uk/exam-arrangements/assessment-information/sqe1-sample-questions/flk2/question40

The best answer is E. The man will be eligible for publicly funded legal representation by the duty solicitor at the police station because he has been arrested on suspicion of a criminal offence, which is one of the qualifying criteria for legal aid. Under the Legal Aid, Sentencing and Punishment of Offenders Act 2012, individuals who have been arrested and taken to a police station for questioning are entitled to free legal advice from a duty solicitor. However, the man will not be able to get a representation order in the Magistrates’ Court, as he will fail the means test. The means test takes into account the income and assets of an individual and determines their eligibility for legal aid based on whether they can afford to pay for legal representation themselves. In this case, the man is a professional football player earning a high income of £180,000 per year, which would likely exceed the threshold for legal aid in the Magistrates' Court. Therefore, even if he passes the interests of justice test, which considers the seriousness of the offence and the potential consequences for the individual, he will not be able to obtain legal aid in the Magistrates’ Court due to his means. However, he may still be able to access legal representation by hiring a private solicitor, but he will have to bear the costs himself.

Question 41[edit | edit source]

FLK2- Wills https://sqe.sra.org.uk/exam-arrangements/assessment-information/sqe1-sample-questions/flk2/question41

The correct answer is E.

Inheritance (Provision for Family and Dependants) Act 1975 requires all claims to be brought within six months of the date upon which a Grant of Representation is first taken out, otherwise the permission of the Court is required.

Question 42[edit | edit source]

Criminal Law https://sqe.sra.org.uk/exam-arrangements/assessment-information/sqe1-sample-questions/flk2/question42

The correct answer is D.

Section 47 of the Offences Against the Person Act 1861 states: “Whosoever shall be convicted upon an indictment of any assault occasioning actual bodily harm (ABH) shall be liable... to be imprisoned for any term not exceeding five years”

The definition of an assault occasioning actual bodily harm (ABH): Actus Reus: the defendant must commit an assault or battery which causes the victim to suffer actual bodily harm. Mens Rea: the defendant must intend or be reckless as to the assault or battery. Note: there is NO need to show that the defendant intended or foresaw actual bodily harm. The offence of ABH can be broken down into three elements: (1) It must be shown that there was an assault or battery. (2) The victim must suffer ABH. (3) It must be shown that the ABH was occasioned by the common assault or battery of the defendant. In this scenario, the recklessness is met when the man approached the woman holding a large knife, which created a risk that the woman would fear immediate unlawful force and the man realised that his actions created a risk that the woman would fear immediate unlawful force, and he proceeded with his actions regardless.

Question 43[edit | edit source]

Wills https://sqe.sra.org.uk/exam-arrangements/assessment-information/sqe1-sample-questions/flk2/question43

The correct answer is A.

The man's will was revoked by destruction when he tore it up into four pieces. This act of physical destruction of the original will, done with the intention of revoking it, means that the will is no longer legally valid. This is because the destruction of the will is considered a deliberate and unequivocal act of revocation, regardless of the testator's later regrets or changes of mind. Therefore, the man's estate will pass in accordance with the rules of intestacy, unless he makes a new will or a valid codicil to the original will.

Question 44[edit | edit source]

Land Law https://sqe.sra.org.uk/exam-arrangements/assessment-information/sqe1-sample-questions/flk2/question44

The correct answer is B.

The alterations covenant in the lease provides that the tenant may not make alterations to the building except for non-structural alterations. The tenant's proposed alterations to the external wall of the building would involve structural alterations, as they would require the creation of new apertures in the wall to accommodate windows. Therefore, the tenant would need to obtain the Landlord's consent to make these alterations.

However, the lease does not require the Landlord to give reasons for withholding consent. Additionally, the alterations covenant contains an absolute prohibition against structural alterations, meaning that the Landlord can withhold consent to the tenant's proposed alterations without having to provide any reasons. As such, the Landlord has the right to withhold consent, and the tenant would need to seek alternative options or negotiate with the Landlord to come to an agreement on the proposed alterations.

Question 45[edit | edit source]

Solicitors Account https://sqe.sra.org.uk/exam-arrangements/assessment-information/sqe1-sample-questions/flk2/question45

The correct answer is D.

The firm of solicitors should transfer £20,000 from the executors’ client account ledger to the beneficiary’s client account ledger because the beneficiary has given written instructions to use their legacy to repay the £2,000 incurred in surveyor’s fees and searches and to retain the balance of £18,000 to be used as part of the purchase price. As the beneficiary is entitled to the £20,000 legacy under the will, the firm has a duty to follow the beneficiary’s instructions and transfer the money accordingly.

The firm should then transfer £2,000 to the firm’s business bank account because this amount has been billed to the beneficiary for surveyor’s fees and searches. The beneficiary has instructed the firm to deduct this amount from their legacy, and the firm has a duty to follow these instructions. Once the deduction has been made, the remaining balance of £18,000 can be transferred to the beneficiary’s client account ledger and used as part of the purchase price for the property.

Question 46[edit | edit source]

Wills

https://sqe.sra.org.uk/exam-arrangements/assessment-information/sqe1-sample-questions/flk2/question46

The best answer is E.

The condition for the grandson to inherit the trust fund was that he qualifies as a solicitor in England and Wales. Since the grandson died before fulfilling this condition, he cannot inherit the trust fund. Therefore, the trust fund would revert to the man (the settlor), as he was the one who established the trust and intended for the trust fund to ultimately pass to his grandson only if he met the specified condition.

Question 47[edit | edit source]

Land Law: Proprietary Rights: essential characteristics of easements https://sqe.sra.org.uk/exam-arrangements/assessment-information/sqe1-sample-questions/flk2/question47

The best answer is A.

In this scenario, it's important to understand the legal concepts involved: lease, licence, easement, and Section 62 of the Law of Property Act 1925.

Lease: A lease is a contractual agreement between a landlord and a tenant, granting the tenant exclusive possession of the leased premises for a certain period of time, usually in exchange for rent.

Licence: A licence is a permission granted by a landowner to another party to use the land for a specific purpose. Unlike a lease, a licence does not confer exclusive possession of the land.

Easement: An easement is a legal right to use someone else's land for a specific purpose. It grants a non-owner the right to use a portion of another person's property for a particular purpose. Easements can be created expressly, impliedly, or by prescription.

Section 62 of the Law of Property Act 1925: This section of the law deals with the creation of legal easements and licences. It states that a deed is not necessary to create an easement or licence, but certain conditions must be met.

Now, let's apply these concepts to the scenario:

The woman initially had a licence from the landlord to use the storeroom for storing stock. When the new lease was created, it effectively converted her licence into an easement. This is because the new lease granted her exclusive possession of the shop, which would include the right to use the storeroom, unless explicitly stated otherwise. Even though the provisions of the new lease did not specifically reference Section 62 of the Law of Property Act 1925, the grant of exclusive possession under the lease is sufficient to create an easement. Therefore, the woman likely has an enforceable easement to use the storeroom, as it was effectively converted from a licence to an easement when the new lease was granted. This means she has a legal right to continue using the storeroom for storing stock, and she may be able to prevent the landlord from demolishing it.

Question 48[edit | edit source]

Criminal Procedure: https://sqe.sra.org.uk/exam-arrangements/assessment-information/sqe1-sample-questions/flk2/question48

The best answer is D.

Question 49[edit | edit source]

Wills

https://sqe.sra.org.uk/exam-arrangements/assessment-information/sqe1-sample-questions/flk2/question49

The best answer is C.

Question 50[edit | edit source]

Property Practice

https://sqe.sra.org.uk/exam-arrangements/assessment-information/sqe1-sample-questions/flk2/question50

The best answer is D.

Question 51[edit | edit source]

Land Law https://sqe.sra.org.uk/exam-arrangements/assessment-information/sqe1-sample-questions/flk2/question51

The best answer is A.

Question 52[edit | edit source]

Criminal Law: https://sqe.sra.org.uk/exam-arrangements/assessment-information/sqe1-sample-questions/flk2/question52

The best answer is E.

Question 53[edit | edit source]

Wills

https://sqe.sra.org.uk/exam-arrangements/assessment-information/sqe1-sample-questions/flk2/question53

The best answer is B.

Question 54[edit | edit source]

Taxation https://sqe.sra.org.uk/exam-arrangements/assessment-information/sqe1-sample-questions/flk2/question54

The best answer is C.

Question 55[edit | edit source]

Criminal Procedure https://sqe.sra.org.uk/exam-arrangements/assessment-information/sqe1-sample-questions/flk2/question55

The best answer is D.

Question 56[edit | edit source]

Property Practice https://sqe.sra.org.uk/exam-arrangements/assessment-information/sqe1-sample-questions/flk2/question56

The best answer is A.

Question 57[edit | edit source]

Taxation

https://sqe.sra.org.uk/exam-arrangements/assessment-information/sqe1-sample-questions/flk2/question57

The best answer is E.

Question 58[edit | edit source]

Land Law

https://sqe.sra.org.uk/exam-arrangements/assessment-information/sqe1-sample-questions/flk2/question58

The best answer is B.

The interest created in the transfer is a restrictive covenant, as it limits the use of the land by prohibiting the erection of more than four single-storey houses. Restrictive covenants can be either legal or equitable. Legal restrictive covenants are enforceable at law, usually through a court order, and bind successors in title without the need for privity of estate. Equitable restrictive covenants, on the other hand, are enforceable in equity and require privity of estate to bind successors in title. Here, since the covenant binds successors in title, it is likely to be an equitable restrictive covenant. Therefore, the correct answer is:

Question 59[edit | edit source]

Criminal Law: General defences: self-defence

https://sqe.sra.org.uk/exam-arrangements/assessment-information/sqe1-sample-questions/flk2/question59

The best answer is D.

Question 60[edit | edit source]

Taxation

https://sqe.sra.org.uk/exam-arrangements/assessment-information/sqe1-sample-questions/flk2/question60

B

Question 61[edit | edit source]

Trust Law https://sqe.sra.org.uk/exam-arrangements/assessment-information/sqe1-sample-questions/flk2/question61

The best answer is B.

The personal claim against the girlfriend is likely to succeed if it can be proven that she received the trust money with knowledge of the breach of trust by the man.

In this scenario, the girlfriend had knowledge of several key factors:

The man's inability to afford the transfer of £200,000, given his outstanding debts. The man's status as a trustee and his willingness to unlawfully take money from the trust for his own purposes. She did not inquire about the source of the money, despite being aware of the above facts. Given these circumstances, the girlfriend's knowledge of the breach of trust, combined with her failure to inquire about the source of the money, could be construed as complicity in the breach. Therefore, option A is the most likely correct answer:

A. Yes, because her knowledge made her suspicious but she decided not to enquire about the money.

Question 62[edit | edit source]

Wills and Intestacy: Validity of wills and codicils

https://sqe.sra.org.uk/exam-arrangements/assessment-information/sqe1-sample-questions/flk2/question62

The best answer is C.

A testator must either sign the will or acknowledge the signature in the presence of 2 or more witnesses.

Question 63[edit | edit source]

Land Law

https://sqe.sra.org.uk/exam-arrangements/assessment-information/sqe1-sample-questions/flk2/question63

A legal lease for three years or fewer can be created by oral agreement.

Question 64[edit | edit source]

Criminal Practice: Principles and procedures to admit and exclude evidence: hearsay evidence: grounds for admitting hearsay evidence

https://sqe.sra.org.uk/exam-arrangements/assessment-information/sqe1-sample-questions/flk2/question64

The best answer is C.

Res Gestae is a Latin term which literally translates as “things done”. It means, “a statement made by a person so emotionally overpowered that the possibility of concoction or distortion can be disregarded”. (section 118 CJA 2003) It is now expressly included as a gateway to admit hearsay under section 118 CJA 2003.

Source: https://www.exchangechambers.co.uk/victimless-prosecution-applying-and-resisting-res-gestae/

Question 65[edit | edit source]

Property Practice

https://sqe.sra.org.uk/exam-arrangements/assessment-information/sqe1-sample-questions/flk2/question65

The best answer is C.